Proof by Induction: Prove a(n)=2^(n-1)[n+2]

  • Thread starter Thread starter kmeado07
  • Start date Start date
  • Tags Tags
    Induction Proof
Click For Summary
The discussion focuses on proving the formula a(n)=2^(n-1)[n+2] for the sequence defined by a(0)=1, a(1)=3, and a(n)=4[a(n-1) - a(n-2)] for n≥2 using mathematical induction. The initial step involves proving the base case for n=0, which is straightforward. The conversation emphasizes the use of strong induction, where the assumption is made that the statement holds for all integers less than n. By substituting the assumed values of a(n-1) and a(n-2) into the recurrence relation, the proof can be completed. This method effectively demonstrates the validity of the formula for all n≥1.
kmeado07
Messages
40
Reaction score
0

Homework Statement


Define the numbers a(0),a(1),a(2),...by a(0)=1, a(1)=3, a(n)=4[a(n-1) - a(n-2)]. for n>=2
Show by induction that for all n>=1, a(n)=2^(n-1) [n+2]

Homework Equations


The Attempt at a Solution



proof by induction is not my stong point, and i really don't know where to start on this question. Any help would be much appreciated!
 
Physics news on Phys.org


One nice thing about induction is you always know "where to start"- prove the statement is true for n= 0!

Then you will really want to use "strong induction": If, whenever a statement is true for all k less than n, it true for k= n, then the statement is true for all positive integers.

If the statement is true for all k< n, it is, in particular true for n-1 and n-2: you can assume that
a(n-1)= 2^{n-1-1}(n-1+ 2)= 2^{n-2}(n+1)
and
a(n-2)= 2^{n-2-1}(n-2+2)= 2^{n-3}n

Put those into a(n)= 4[a(n-1)- a(n-2)] and see what you get.
 


thanks for the help!
 
Question: A clock's minute hand has length 4 and its hour hand has length 3. What is the distance between the tips at the moment when it is increasing most rapidly?(Putnam Exam Question) Answer: Making assumption that both the hands moves at constant angular velocities, the answer is ## \sqrt{7} .## But don't you think this assumption is somewhat doubtful and wrong?

Similar threads

  • · Replies 7 ·
Replies
7
Views
1K
  • · Replies 7 ·
Replies
7
Views
2K
  • · Replies 15 ·
Replies
15
Views
3K
  • · Replies 2 ·
Replies
2
Views
2K
  • · Replies 3 ·
Replies
3
Views
2K
Replies
1
Views
2K
  • · Replies 1 ·
Replies
1
Views
1K
Replies
6
Views
2K
Replies
5
Views
2K
  • · Replies 7 ·
Replies
7
Views
2K